- PowerScore Staff
- Posts: 5972
- Joined: Mar 25, 2011
- Sat Feb 20, 2010 12:00 am
#59771
Complete Question Explanation
(The complete setup for this game can be found here: lsat/viewtopic.php?t=26553)
The correct answer choice is (A)
The question stem sets up the following sequence:
W J
Although this rule interrelates the majors and nonmajors, more importantly it ties in the VFJ super-block. If W is assigned to a lower-number bench than J, it follows that W is assigned to a lower-numbered bench than V and F as well:
At this point, you can actually determine that answer choice (A) is correct because since W is assigned to a lower-numbered bench than J, only H remains to be assigned to the same bench as W, meaning that H must be assigned to a lower-numbered bench than F. However, if you do not see that inference immediately, move on to making hypotheticals.
Because the VFJ super-block is again forced into only two possible positions: benches 2 and 3 or benches 3 and 4, as in the previous two questions, you can quickly make the two applicable hypotheticals:
When the above hypotheticals are applied to the answer choices, it is apparent that answer choice (A) is correct. Answer choices (B), (C), and (E) could be true but do not have to be true. Answer choice (D) can never be true.
(The complete setup for this game can be found here: lsat/viewtopic.php?t=26553)
The correct answer choice is (A)
The question stem sets up the following sequence:
W J
Although this rule interrelates the majors and nonmajors, more importantly it ties in the VFJ super-block. If W is assigned to a lower-number bench than J, it follows that W is assigned to a lower-numbered bench than V and F as well:
At this point, you can actually determine that answer choice (A) is correct because since W is assigned to a lower-numbered bench than J, only H remains to be assigned to the same bench as W, meaning that H must be assigned to a lower-numbered bench than F. However, if you do not see that inference immediately, move on to making hypotheticals.
Because the VFJ super-block is again forced into only two possible positions: benches 2 and 3 or benches 3 and 4, as in the previous two questions, you can quickly make the two applicable hypotheticals:
When the above hypotheticals are applied to the answer choices, it is apparent that answer choice (A) is correct. Answer choices (B), (C), and (E) could be true but do not have to be true. Answer choice (D) can never be true.
You do not have the required permissions to view the files attached to this post.
Dave Killoran
PowerScore Test Preparation
Follow me on X/Twitter at http://twitter.com/DaveKilloran
My LSAT Articles: http://blog.powerscore.com/lsat/author/dave-killoran
PowerScore Podcast: http://www.powerscore.com/lsat/podcast/
PowerScore Test Preparation
Follow me on X/Twitter at http://twitter.com/DaveKilloran
My LSAT Articles: http://blog.powerscore.com/lsat/author/dave-killoran
PowerScore Podcast: http://www.powerscore.com/lsat/podcast/